This section is Section 1 of 3.

Speed as well as accuracy is important in this section. Work quickly, or you might not finish the paper. There are no penalties for incorrect responses, only marks for correct answers, so you should attempt all 50 questions. Each question is worth one mark.

You must complete the answers within the time limit. Calculators are NOT permitted.

Good Luck!

Note – if press “End Exam” you can access full worked solutions for all past paper questions

11% of nurses across the country would like to leave the profession. However, this figure rises to 39% amongst nurses who are working alternating shift systems, which mix day, evening and night shifts. It was found that many nurses preferred one type of shift most of the time, whether that involved working early, late or through the night. Employers should urgently review staffing arrangements to avoid the compulsory, alternating patterns. This would make nurses’ working hours more compatible with their social and family lives. A further benefit would be a reduction in the cost of training new nurses to replace those leaving.

1. What is the best expression of the main conclusion of this argument?
  • 1
    1

    Explanation

    The correct answer is A.

    This argument addresses the issues around shift patterns worked by nurses. It talks about how many nurses prefer one type of shift, no matter what the time, therefore employees should be able to avoid compulsory alternating shift patters. As this would be better for their lives outside of work. Looking at the answer options C and E are stated points in the argument, D is not raised in the text, leaving options A and B. Although both are valid conclusions if we use the therefore test, it is clear that answer option A is the main valid conclusion for the argument.

    Post Comment

    To play a football pools game, participants must select four matches from the fifty on the coupon. Points are scored depending on the result of each match as follows:

    If the home team wins, the participant scores           1.0 point

    If the away team wins, the participant scores            1.5 points

    If the match is a draw, the participant scores            3.0 points

    2. Which of the following total scores for the four matches is not possible?
  • 0
    0

    Explanation

    The correct answer is C.

    The totals other than C can be made as follows: A=1+1.5+3+3=8.5, B=1.5+1.5+3+3=9, C=1+3+3+3=10, D=1.5+3+3+3=10.5.

    Post Comment

    Some recent films have been very expensive to make, but have not been the big box-office hits that would have justified the expense. At the same time, there have been films made very cheaply which have been received with both huge critical and popular acclaim. Indeed, some directors who have made successful low-budget films have gone on to make unsuccessful but expensive films. It is obvious then that if directors want to make popular films, they should stick to low budgets.

    3. Which one of the following is the best statement of the flaw in the argument above?
  • 0
    0

    Explanation

    The correct answer is C.

    The argument is that directors should keep film budgets low if they wish for them to be popular. A is not a flaw as the argument refers to popular acclaim as well. B is irrelevant as the argument refers to the actual budget/cost of the film rather than the original budget. D may be true, but the argument implies that low-budget films are often more popular than expensive films, and the cost comes before you know whether the film will be popular or not. Whether or not the public know the cost of the film is irrelevant, so E is not a flaw. Hence C is the flaw, as factors other than the cost could affect whether a film is popular.

    Post Comment

    It is clear that the world is getting warmer. The only thing that could slow down this process of global warming, and avert ecological disaster, would be an immediate switch to green technologies. If global warming continues at the present rate, some species will be unable to live in the warmer conditions. They will need to move to cooler places than they now occupy, for example closer to the poles, or higher up mountains. Not all of these species will be able to reach cooler climates, because of the long distances they would have to travel. Some of those that could move would end up in overcrowded habitats, which would make successful breeding impossible.

    4. Which one of the following can be drawn as a conclusion from the above passage?
  • 0
    0

    Explanation

    The correct answer is E.

    The passage speaks about the ecological disaster that will occur as the world gets hotter. The text speaks around the issues of the rate at which global warming is occurring, and speaks about the effects of this on different species that request cooler environments. Looking at the answer options, A is a point raised in the text, where as B is a point alluded to in the text but not explicitly said. C is not explicitly said that this is the case, as it is specifically refers to endangered species, which are not mentioned in the text. Option B, is untrue, as contradicts the initial argument in the text, that we could slow down the process of global warming with the immediate switch to greed technologies. This leaves us with answer option E, which is the only valid conclusion in the answer options.

    Post Comment

    Cannabis smoking is widespread, even though it is illegal. It is also more dangerous to health than most people realise. If we want to reduce the use of cannabis, it should be legalised. People do not turn away from dangerous habits unless they are fully aware of the damage these habits can cause. We should have learnt this lesson from the example of tobacco smoking, which has declined dramatically because people have seen that it results in deaths from lung cancer.

    5. Which one of the following is an assumption underlying the above argument?
  • 0
    0

    Explanation

    The correct answer is B.

    The argument is that to reduce the risk of cannabis it should be legalised because it will make people more aware of its dangers to health. The statement given in A actually contradicts what is said in the last sentence of the passage, which implies that people are aware due to lung cancer deaths. C cannot be an assumption as the argument is for legalising cannabis. Research is not alluded to in the passage so D is not an assumption. E is not an assumption because the argument is that people are less aware of the dangers of cannabis whilst its use is illegal, not that it being illegal is a reason for its use. Therefore C is the assumption, as for the argument to be valid it must follow that legalising cannabis will make people aware of its dangers, so the converse must be true.

    Post Comment

    To encourage themselves to lose weight, Alma, Brigit, Cara, Danni and Evelyn each put £10 into a pot. They weighed themselves when they began and again at the end of each week. The one who had lost most weight after 4 weeks won the £50. The following chart shows their progress:

    6. Who won the £50?
  • 1
    0

    Explanation

    The correct answer is E.

    Subtract the weight after week 4 from the starting weight for each person: Alma=71-64=7, Bridget=67-62=5, Cara=63.5-57.5=6, Danni=66.5-60=6.5, Evelyn=69-61=8. So the answer is Evelyn.

    Post Comment

    Below is a list of the Angevin and Plantagenet kings of England and the dates of their reigns.

    7. Which of the following bar charts represents a distribution of the lengths of the reigns of these kings?
  • 2
    1

    Explanation

    The correct answer is E.

    The lengths of reign in years for each king in order from top to bottom in the table are: 35, 10, 17, 56, 35, 20, 50, 22. In the 0-15yrs category there is 1 king (whole reigned for 10yrs). In the 16-30yrs, there are 3 kings (reigns of 17, 20 and 22 yrs). For 31-45 there are 2 kings (2 kings both for 35yrs) and for 46-60yrs there are 2 kings (whole ruled for 50 and 56yrs). So the heights of the bars from left to right should be 1, 3, 2, 2, which is given by answer E.

    Post Comment

    A man drives off from his house and travels through the village at a constant speed. As he leaves the village behind, his speed increases to a new constant value which he maintains until he reaches a steep hill which slows him down. At the top of the hill the driver stops for a few minutes to admire the view. He realises that he has left his picnic basket at home and so drives rapidly back along his route to collect it.

    8. Which one of the following graphs best represents the driver’s distance from home throughout his journey?
  • 0
    0

    Explanation

    The correct answer is D.

    Initially the man drives at a constant speed, so the relationship between time and distance will be proportional, ie a straight line with a postive gradient; eliminate C as the first part of the graph is horizontal. His speed then increases, so the gradient will suddenly increase (the line will be steeper); eliminate B as the line becomes less steep. He then slows down upon reaching a steep hill, so the gradient will decrease; we cannot eliminate anymore graphs at this point. The driver then stops, so the graph will be horizontal; eliminate E because the graph has a negative gradient at this point. He then begins driving back home, so the gradient will be negative as he is getting closer to home; this is shown by graph D but not A, so D is the answer.

    Post Comment

    If children are sitting in rows in a classroom, the teacher can have eye contact with all of them while she is explaining something to them. This is not always possible if they are sitting in groups around tables. Also, when they look up, instead of seeing the child opposite in a group and being tempted to talk, they see the teacher. So, sitting in rows helps children to concentrate better on their work and should therefore be the standard arrangement in every school classroom.

    9. Which one of the following, if true, would most strengthen the above argument?
  • 1
    0

    Explanation

    The correct answer is E.

    The argument is that children in classrooms should be sat in rows. B gives a disadvantage of sitting in rows and groups so does not strengthen the argument that rows are better. C gives an advantage of sitting in groups, which the passage is arguing against, hence it weakens the argument. D only highlights an opinion therefore does not most strengthen the argument. E does strengthen the argument by giving an advantage of sitting in rows (over groups) and therefore is the answer. A does give an advantage of sitting in rows, but as the same is said about sitting in groups, it does not strengthen the argument as much as E.

    Post Comment

    “There are more police officers than ever before in the history of this country. We have more police officers and more community support officers. That is why, last week, crime was down. Crime is down 30 per cent.”

    10. Which of the following states the flaw in the argument above?
  • 0
    0

    Explanation

    The correct answer is C.

    The argument is that crime was down because we have more police and community support officers. But the argument does not give evidence for the casual relationship they have stated. The crime could have fallen regardless of whether or not there were more officers, which is highlighted by C. A is not a flaw because the argument still states that there were more police officers than previously. B is irrelevant as the method of measuring crime is not mentioned. D is also irrelevant because the argument focuses on the fact that crime has fallen, not that this trend will continue. E, again, is irrelevant because the argument refers to the level of crime not crime detection.

    Post Comment

    The differences between human beings are sometimes more striking than the similarities. There is a minority of people who, driven by a competitive urge to be the first or the best, undertake ambitious expeditions involving severe physical deprivations, high risk of fatality, and extreme isolation. Compare these people to the majority who prefer an easy, risk free life of comfort and we can see that one human being can be as different from another in their behaviour as two different species of animal. Any attempt to form general rules about human behaviour needs to bear this variation in mind.

    11. Which one of the following best expresses the conclusion of the passage above?
  • 1
    0

    Explanation

    The correct answer is C.

    The text speaks about the differences between individuals who are competitive and risk taking in their behaviour, in comparison to those who prefer a risk free life of comfort. It speaks about how the various between mind is something that needs to be thought about when creating general rules about human behaviour. Looking at the answer options, A is something stated in the passage, while B is an irrelevant point not raised in the text. D is again a stated fact from the text, and E is something that is completely irrelevant to the passage. This leaves us with the only possible answer option, C.

    Post Comment

    Jars of Bestcaff coffee weighing 200g are normally £2.50 at Kostless and Savemore, but both supermarkets have a special offer this week:

    12. By how much per kilogram is Kostless’s offer better value for money than Savemore’s?
  • 0
    0

    Explanation

    The correct answer is A.

    Cost for 200g at Kostless: 2.50-0.80=1.70; therefore for 1000g (which=200gx5) it would cost 1.70×5=£8.50. If we want 1kg from Savemore, we need to find the weight so that 25% extra would give us 1kg in total. Let this weight be ‘y’: y x 1.25 = 1000g. Therefore y=1000/1.25=1000×4/5=800g. At £2.50 per 200g, this would cost: 2.50×4=£10. Therefore, the cost difference per kg would be: 10-8.50=£1.50.

    Post Comment

    The table shows the areas of the countries of the European Union.

    13. What is the minimum number of countries needed to make up at least 50% of the total land area?
  • 0
    0

    Explanation

    The correct answer is A.

    Because we want the smallest number of countries, we need to use the countries with the largest land area. So, start with the largest and second largest, add together their land areas, then continue adding the land area of the next largest country until you have met/exceeded 50% of the total land area, which is 4422773/2=2211386.5 sq km. But since the smallest possible number of countries it could potentially be is 5 (as this is the smallest option out of the answer choices), we can start by adding together the land areas of the 5 largest countries and seeing if this exceeds 2211386.5 sq km. France has the largest land area, then Spain, Sweden, Germany and Finland. The calculation would be: 643548+504782+449964+357021+337030=2292345, which is greater than 2211386.5. Therefore the minimum number of countries needed is 5.

    Post Comment

    There are seven books on my shelf, which are of various sizes. When I look at them straight on I see the following picture:

    I now look at them from the left-hand end (in the direction of the arrow).

    14. Which one of the following diagrams is not a possible view when seen from the left-hand end?
  • 0
    0

    Explanation

    The correct answer is C.

    From the view when looking that the books in the book shelf straight on, we can only determine the relative heights of the books, but not their widths. Hence when we look at them from the left, we can determine which of the views are possible on the basis of the different heights, but not the widths. From the view straight on, we can see that we should be able to see the top edges of at least 4 books, (highlighted in blue) simply because each one is taller than the previous ones. Therefore, any answer options where we can see these 4 books of different heights on the leftmost edge of the view is possible, as shown for options A and B. However, in answer option C, we only see 3 books on the leftmost edge, and therefore this is not a possible view.

    Post Comment

    Similar effects have similar causes. The problem is deciding when one set of effects sufficiently resembles another to infer a similar cause. You say that the universe resembles a man-made object like a house, so its cause must also be similar, an intelligent designer, resembling human beings. But is the universe so like a house? It seems to resemble an animal or vegetable just as closely, so why should its origin not lie in reproduction or some vegetative process? The universe contains many classes of effects and objects, all with their different causes. There seems to be no compelling reason for concluding that the universe as a whole resembles one of these classes of effects or objects more closely than another.

    15. Which one of the following is a conclusion that can reliably be drawn from the above reasoning?
  • 0
    0

    Explanation

    The correct answer is B.

    This passage talks about how the universe resembles man-made objects, and speaks around this abstract interpretation. The text goes on to speak about how it doesn’t resemble any particular class, effect, or object more than another. Looking at the answer options, both A and C are based on points raised in the text, and therefore not valid. While answers D and E are untrue based on the points raised in the text. Therefore, the only valid answer option is B.

    Post Comment

    At first glance, there is no phenomenon creepier than crop circles – huge geometric designs that appear in fields overnight seemingly without cause, explanation or reason. UFO enthusiasts are quick to supply all three, by attributing the patterns to alien visitors seeking to communicate with humans, using laser beams or other advanced technology to flatten the crops. These fanciful claims can fortunately be dismissed thanks to two quite mundane observations. Firstly, ordinary Earth-born individuals have repeatedly demonstrated how to produce the effects with nothing more elaborate than a rope and a plank. Secondly, there is a clear parallel for the phenomenon in the form of urban graffiti. Sneaking into fields to make crop circles is simply a rural variation of a widespread and well-documented human urge.

    16. Which one of the following is an implicit, underlying assumption that is made by the above argument?
  • 0
    0

    Explanation

    The correct answer is A.

    The argument is that crop circles are not caused by aliens/UFOs because the crops circles can be easily made and because it is an example of human graffiti. However, just because the arguments give ways that crops circles can be formed, UFO landings could still remain an explanation, i.e. the argument does not disprove the theory that crop circles are caused by UFO landings. Therefore the argument assumes that, although the possible explanation of alien activity exists, we should accept the more plausible explanation that the crop circle effects have been created by humans. This is highlighted by answer A.

    Post Comment

    When you install a burglar alarm in your home, you do so presumably because the cost of the alarm is less than the expected benefits. The cost of a burglary, if one occurs, is likely to be much higher than the cost of fitting an alarm. A clearly visible box on an outside wall of an alarmed house alerts and discourages potential thieves. Because your house is less likely to be burgled, your neighbour’s house may be more vulnerable. Therefore, while there may be little impact on overall crime levels, expensive security devices reduce crimes against those with the devices but increase crimes against those without.

    17. Which one of the following, if true, would most weaken the above argument?
  • 0
    0

    Explanation

    The correct answer is C.

    The argument is that security devices reduce crime against those who have them, whilst increasing crime against those without them. C weakens the argument by implying that burglars are more likely to steal from houses with alarms than those without, because having an alarm suggests the house has valuable items in it, worthy of stealing. A strengthens the argument by suggesting that a larger proportion of houses without alarms are burgled compared to those with alarms. B also strengthens the argument, by showing that intruder alarms seemed to have deterred house burglars. D and E are irrelevant, as they do not allude to the likehood of houses with security devices being burgled compared to those without these devices.

    Post Comment

    Dates may be written in an eight digit form. For instance, 19 January 2005 may be written 19-01-2005.

    18. In what year will the next date occur for which all eight digits are different?
  • 0
    2

    Explanation

    The correct answer is D.

    Post Comment

    This is the view from above of the large three-dimensional logo situated in the forecourt of the head office of Justin Thyme Enterprises:

    It was sculpted from a single cube of marble, and has a uniform cross-section from top to bottom.

    19. Which of the following is not a possible side view of the logo?
  • 0
    0

    Explanation

    The correct answer is B.

    A is the view from the right side, C from the bottom, D from the top and E from the left. Therefore B is the answer.

    Post Comment

    The table below, from a newspaper, shows how much money ($) you will need at various points in the future to buy items costing $1000 today, depending on the annual rate of inflation.

    Raoul has just bought a new car for $10 000. He expects this car to last him 8 years, when he will buy another equivalent model. He puts $10 000 into a bank account ready to pay for the next car. The price of cars rises with inflation at 8% and his savings only earn interest of 6%.

    20. How much will he need to add to his savings when he needs to buy his new car?
  • 1
    1

    Explanation

    The correct answer is B.

    Post Comment

    It would be a good idea if the police force was separated into two divisions, one for proper policing i.e. protecting citizens and upholding law and order, and the other for road traffic. This second division could be combined with traffic wardens as basically they are doing a similar job: obtaining money from road users. Perhaps then there would be a proper focus put on catching criminals and protecting the law-abiding public, and we would have a police force doing the job they were actually intended to do.

    21. Which one of the following best describes the flaw in the above argument?
  • 0
    0

    Explanation

    The correct answer is A.

    The argument is for the idea of dividing the police force into two groups, one for protecting citizens and the other for road traffic. However, it fails to acknowledge that they may be some cross-over between the two groups’ roles, which is highlighted by A. B is not a flaw because the argument reason for a better focus on catching criminals and does not suggest that this is currently non-existent, only that it may be minimal. C is not a flaw, because even though they have different powers, there is not a reason given against them working together. D cannot be a flaw because the passage does not do this, but rather mentions both. E may be implied by the final sentence of the passage, but this does not invalidate the argument so is not a flaw.

    Post Comment

    If you practise playing a musical instrument, your playing will get better. I have not practised playing the piano, so my playing has not got better. I have an exam in two month’s time. So I must practise every day if I want to do well.

    22. Which of the following most closely parallels the reasoning in the argument?
  • 0
    0

    Explanation

    The correct answer is B.

    If you practise (playing a musical instrument), you will get better at X. I have not practised X, so I have not got better at X. I have an exam coming up so I must practise if I want to do well. answer B parallels this, just substitute X for sport and ‘practising’ for ‘working hard’.

    Post Comment

    Legislation is needed in order to prevent the abuse of genetic testing by insurance companies. New genetic tests may reveal how likely an individual is to suffer from certain debilitating or fatal conditions in later life. Without legislation, insurance companies might make such tests compulsory and make those at risk pay more for insurance. This would be unfair. We do not choose our genes; we are born with them. People should not be penalised for things over which they have no control.

    23. Which one of the following best illustrates the principle underlying the above argument?
  • 0
    0

    Explanation

    The correct answer is D.

    Post Comment

    As part of a journey, I must travel from Bowbridge to Downland. I can choose either the motorway (known locally as ‘the road to hell’) or the old road known as Buttercup Way. On the motorway I average 60 mph, on the old road I average 40 mph. I can change from one to the other at Clatterton where they cross.

    The distances by the two roads are as follows:

    24. If I take the fastest route, what will be the time saving over the slowest route?
  • 0
    0

    Explanation

    The correct answer is C.

    Calculate whether the motorway or Buttercup way would be the quickest for both Bowbridge to clatterton and from clatterton to downland. Do this by dividing the distance by the speed to get time. For Bowbridge to clatterton, on the motorway it would take: 35/60=7/12 hrs, whereas on Buttercup way it would take 30/40=9/12 hrs. 7/12<9/12, therefore it would take less time to travel on the motorway from Bowbridge to Clatterton. For Clatterton to Downland, on the motorway it would take 7/12 hrs again, whereas on Buttercup way it would take 20/40=6/12 hrs. 7/12>6/12, therefore it would take less time on Buttercup way from Clatterton to Downland. Therefor the fastest route would be on the motorway for the first part of the journey and then on Buttercup way for the second part, which would take: 7/12+6/12=13/12hrs. The slowest route would be the opposite (Buttercup way first and then the motorway) which would take: 9/12+7/12=16/12hrs. The difference between these is: 16/12-13/12=3/12=0.25hrs, which is 15 minutes.

    Post Comment

    Various shapes may be made by overlapping two transparent squares, for example the dark shaded hexagon shown below:

    25. Which one of the following shapes cannot be made as the overlap of two transparent squares?
  • 0
    0

    Explanation

    The correct answer is B.

    This questions requires that you visualise the two squares in different arrangements that produce the overlaps shown in the answer options. It is useful to look out for parallel sides and 90-degree angles in the overlapping shape as those can help indicate the placement of the two transparent squares. As shown, answer options A, C, D and E are all possible, and therefore the correct answer is option B.

    Post Comment

    This table shows how household sizes in Great Britain changed between 1961 and 1991.

    26. What was the change in the percentage of households with fewer than 3 people between 1971 and 1991?
  • 0
    0

    Explanation

    The correct answer is D.

    We only need to consider the two rows with household sizes of 1 person and 2 people, and only the columns for 1971 and 1991. In 1991, 27+34=61% households had 1 or 2 (fewer than 3) people. For 1971, this value was 18+32=50%. Therefore the percentage change was: 61-50=11%.

    Post Comment

    Britain’s property market is in the midst of a full-scale slump with house prices falling at their fastest rate in 15 years, according to a new study. But why is there an assumption that continuing rises in house prices is a good thing? Why not see the fall in house prices as a welcome change? House prices as a percentage of income are grossly overpriced. Low interest rates and cheap mortgages have led to the property market, particularly in London, being overvalued. Surely this will make houses more affordable. First time buyers will be happy beneficiaries of a housing slump. Others finding a silver lining in the gloom are those in rural communities for whom a house had seemed simply out of reach.

    27. Which one of the following is the main conclusion of the above argument?
  • 0
    0

    Explanation

    The correct answer is E.

    This passage talks about the perception of the property market in Britain, and how house prices are falling at the highest rate in 15 years, yet we perceive this as a bad thing. It speaks about how the property has becoming overpriced and this drop in property prices and cheap mortgages rates will be beneficial to first time buyers. Looking at the answer options, answer A, C and D, although true, are all valid and all contribute to the overall view of the conclusion, yet, are not an overall main conclusion of the text. While option B is not a valid conclusions as is just a fact stated in the text. This leaves us with answer E, which concludes the main overall view of the text. In addition to this, answer options A, C and D, if true, would all contribute to concluding that answer option E is the main conclusion of the text.

    Post Comment

    The average life expectancy in working class areas of London was significantly higher than in similar areas in other major cities in the first half of the nineteenth century. In London during this time drinking tea had become a part of daily life amongst the poor. The reason for this was a combination of availability and cost. Boats bringing tea to the country docked in London and a certain amount was sold off directly to local markets. At this time in the rest of the country tea was still regarded as a luxury, consumed only by the better off.

    28. Which one of the following conclusions is best supported by the passage above?
  • 0
    0

    Explanation

    The correct answer is C.

    The author speaks about different working class areas and how those in London drank tea during the first half of the nineteenth century, while elsewhere in the country tea was reserved as a luxury enjoyed by those who were wealthier. The passage is relating the middle class in London drinking tea, to the increase in life expectancy, in comparison to the working class from other major cities. Looking at the answer options, both answers A and B are quite drastic conclusions, and are reliant on a large amount of assumption and a direct causative relation, rather than that of possible correlation. While looking at answers D and E, which are both irrelevant and not spoken about in the text at all. This leaves the only realistic conclusion, that is supported best by the text as option C. Which also fits to the possible correlation that this article is referring to. For clarity on these questions, think about the difference between causation and correlation.

    Post Comment

    A recent study by a drugs company found that the amount of money spent on a specific pill used for the treatment of Alzheimer’s disease in the UK showed wide regional variation. Alzheimer’s is a condition associated with the ageing process. In some areas the expenditure was as low as £1 per head of the population over 65 whereas in others it was as high as £10 per head. These pills are the most effective treatment for this condition that is currently available. Therefore, this is another example of ‘post-code lottery’ where the effectiveness of treatment for a health condition will depend upon where you live.

    29. Which one of the following is an assumption underlying the above argument?
  • 0
    1

    Explanation

    The correct answer is C.

    Post Comment
    anon Medicmind Tutor

    Wed, 27 Sep 2023 08:33:12

    why is the answer C?

    All the teams in a local football league have now played each other once this season. Below is the current league table:

    30. What was the result of the match between United and County?
  • 0
    0

    Explanation

    The correct answer is A.

    Post Comment
    T Medicmind Tutor

    Thu, 27 Oct 2022 19:02:48

    Overall there are only two draws for County and one draw for United (Utd) and one draw for city. Hence, city must draw with County and Utd must draw with City. Therefore, B,C,E can be excluded. Now, since Utd scored 2 goals and won two games, both games must have been 1-0, hence Utd must have drawn 0-0 in it's draw and lost 0-1 in its lost. Utd cannot draw with County 1-1, as Utd scored 2 goals total and won two games (which it must have been 1-0 each game). As a result, Utd drew with County 0-0 and your answer is A. Answer: A

    T Medicmind Tutor

    Thu, 27 Oct 2022 19:03:51

    Sorry I meant, Utd draws with County and County draws with City.

    In a modified form of a popular game, each of two players at any turn may put either ‘O’ or ‘X’ as she chooses in any blank square of a 3 by 3 grid. She wins if she thereby completes a line of three identical symbols. A player is faced with this situation:

    31. Which one of the following moves ensures that she can win at her next move?
  • 0
    0

    Explanation

    The correct answer is C.

    C is the answer because if the opponent puts either an ‘X’ or and ‘O’ in any of the remaining squares, the player will be able to form a line of 3 identical symbols and therefore win.

    Post Comment

    The following table shows the results of a test on smokers trying to quit. Of 240 people, half were given nicotine tablets and half were given a placebo (a substance containing no medication).

    The researchers assumed that: of the smokers given nicotine tablets, the same proportion would have quit without using them as would have quit using the placebo.

    32. How many of the smokers that were given the nicotine tablets can the researchers conclude had quit due to taking the tablets?
  • 0
    0

    Explanation

    The correct answer is B.

    Of the people who took nicotine, 56 succeeded in quitting. However, 24 people who took the placebo also quit. Since there were the same number of people (120) in both the group taking nicotine and the group taking the placebo, we can assume that out of the 56 people who quit after taking nicotine, 24 of them quit due to the placebo effect. Therefore, the number of people who quit due to the nicotine tablets was: 56-24=32 people.

    Post Comment

    In a high speed police chase, police drivers sometimes lose sight of the car they are chasing. But that’s no reason to give up. All they have to do is take the first left, and then the first left again. Pursued drivers are likely to keep taking turns as they come up, as this seems instinctively like the best way to lose someone who is following them. They are not going to turn right as this involves waiting and checking for oncoming traffic. Therefore take a couple of left turns and the police car will soon have their criminal in sight once again!

    33. Which of the following, if true, would strengthen this argument?
  • 0
    1

    Explanation

    The correct answer is C.

    The argument is that there is a method police can use to catch criminals in a high speed chase, involving taking left turns as they appear, and that this will allow them to catch the criminals. C strengthens the argument by stating an advantage that police have over the criminals, making the argument that the police will catch up with the criminals more likely. A may weaken the argument by implying the opposite to C. B neither strengthens nor weakens the argument as it has already been stated that drivers do this in real life. D may actually weaken the argument by suggesting that the police would require helicopter assistant to gain sight of the criminals they are catching, rather than being able to catch them alone using the left-turn method given in the passage.

    Post Comment

    The museum has decided to spend money in the hope of staying profitable and, therefore, remaining open. It has opened an expensive new front hall, which is very big and well planned, and will undoubtedly impress new visitors. For serious students, however, it has nothing to offer that was not there before. All the scholars who want to do so already use the museum, they will continue to come however artistic the displays may or may not be. So, it was a waste to spend so much money on appearances, the money should have been spent on improving conditions for those who come to study.

    34. Which one of the following is the best statement of the flaw in the above argument?
  • 0
    1

    Explanation

    The correct answer is C.

    The argument is that the money spent to help the museum stay profitable should have been used to improve conditions for those who already come to the museum to study, rather than on building the new front hall to attract new visitors. For this argument to be valid, it must follow that more profit would be made by improving conditions for existing visitors than by building the new front hall. C recognises this flaw: if the visitors whom the conditions are improved for already come to the museum, no more money will be made from there attendance.

    Post Comment

    England is set to become the most crowded country in Europe as its population grows by a third over the next 50 years, according to official projections. There are currently some 50 million people in England, but in 50 years this could hit 68 million – 1,349 for every square mile. Opposition political parties, who have obtained the data from the Office for National Statistics, demanded restrictions on migration levels. The opposition spokeswoman for immigration said “This demonstrates the real pressure public services are being put under as a result of the failed immigration policy of the present government, which the electorate are increasingly recognising.” Continued unrestricted population growth will lead to extremist political parties benefiting at the ballot box.

    35. Which one of the following best expresses the main conclusion of the above argument?
  • 0
    0

    Explanation

    The correct answer is A.

    The passage discusses how population growth in England, caused by unrestricted immigration, if continued will lead to extremist political parties gaining a higher vote. The argument outlines some reasons as to why, with facts and figures to support their argument in the text. Looking at the answers, E is completely irrelevant. While answers B and C are based on points and opinions raised in the argument, however they aren’t full conclusions. They do support answer option D, which although is a valid conclusion to the passage, can still be used to support the main conclusion, A. To see between the two which is the main conclusion and which is not use the therefore test. A therefore D doesn’t work, but D therefore A does, meaning that answer A must be the main conclusion.

    Post Comment

    When my father ran a pizza shop he sold pizzas for a fixed amount plus an amount proportional to the ingredients used. A regular pizza uses twice the ingredients of a mini one and a large pizza twice those of a regular one.

    36. Which one of the following price scales could he have used for mini, regular and large pizzas?
  • 1
    0

    Explanation

    The correct answer is E.

    Form 3 simultaneous equations, one for the cost of each size pizza. Let y be the fixed amount and let x be the amount proportional to the ingredients used on the mini pizza. Mini: y+x, Regular: y+2x (as it has 2 times the amount of ingredients as the mini) and Large: y+4x (as it has 2 times the amount of ingredients as the regular). To find the answer, equate each simultaneous equation to its corresponding price, (e.g. for answer A: y+x=0.49, y+2x=1) and see if the solutions work for all three equations, in which case you will have found the answer. The only trio of prices that give such solutions are in answer E: y+x=2, y+2x=3, subtracting the 2nd equation from the 1st gives x=1 and substituting x=1 into the first equation gives y=1. Then see if these solutions work for the 3rd equation by substituting them into: y+4x, which gives a value of 5, which is the 3rd price given in E. Hence E is the answer.

    Post Comment

    I often order items from a mail order catalogue. The charges for postage and packing (to be added to the order value) are as follows:

    Order value: less than £25.00 – £4.50

    £25.00 to £49.99 – £8.50

    £50.00 to £74.99 – £12.00

    £75.00 to £99.99 – £15.00

    £100.00 and over – £17.50

    Just before sending off my latest order, I noticed that the current catalogue offers half price postage and packing for orders of 8 or more items. My order was for 7 items, with an order value of £74.60. I could not find anything else I wanted, so I just added the cheapest item in the catalogue (priced at £1.25) to my order.

    37. By how much did adding the extra item to my order reduce the total cost?
  • 0
    0

    Explanation

    The correct answer is A.

    Without adding the extra item to the order, the total cost would have been: 74.60+12.00=86.60 . When adding the extra item, the order value would increase to 74.60+1.25=75.85, and the postage and packaging value would increase to 15.00, however because she now has 8 items this value would be halved, meaning the postage and packaging would cost 7.50.I In total this gives: 75.85+7.50=83.35. The difference between the total cost with the extra item compared to without therefore would be: 86.60-83.35=3.25 .

    Post Comment

    John wishes to have 500 copies of a brochure printed and is offered a choice of two price structures:

    1. An equal cost for each brochure printed, regardless of number.

    2. An initial fixed cost followed by a lower cost than structure 1 for each brochure.

    38. Which one of the following sets of information would be sufficient for him to be able to decide which of the two price structures would be least expensive?
  • 0
    0

    Explanation

    The correct answer is A.

    We can assume the number of brochures as X to describe the piece of each structure:

    • Price structure 1) cost = X (Y) (Y is cost per brochure)

    • Price structure 2) cost = W + X(Z) (W is the fixed price) (Z is cost per brochure). Z is smaller than Y

    It is evident that:

    • Due to W, Price structure 2 will begin as most expensive

    • As Z is smaller than Y and as the number of brochures rises, the price of 1) will increase at a larger pace than 2)

    • Price structure 2) will be relatively less expensive compared to 1) as the number of brochures increases until they arrive at a point where they are the same (Equiv)

    • Price structure 2) will be cheaper above the point of Equiv

    Point Equiv will allow us to tell which price structure is cheaper. For example, if this point is greater than 500, 1) will be cheaper, but on the other hand if it is less than 500, then 2) will be cheaper.

    Post Comment

    Many people who live near to mobile phone masts are worried that radiation from these masts will damage their health. Mobile phones themselves have also been suspected of emitting dangerous levels of radiation, and a report recommended that children should be discouraged from making non-essential calls. Emission of radiation from the masts is much lower than that from a mobile phone. A few seconds making a phone call is comparable to 24 hours of exposure from a mast. Radiation from both these sources is below the international guidelines as to what is a dangerous level, but no one can yet be certain if these guidelines accurately reflect the level of risk.

    39. Which one of the following can be drawn as a conclusion from the above passage?
  • 0
    0

    Explanation

    The correct answer is E.

    The text discusses the worries of many who live near mobile phone masts with regards to the radiation and possible damage to health caused by this. The text speaks about how tests have show the emissions are well below international guidelines from what could be considered dangerous, yet no one can be certain if these guidelines accurately reflect the real risk that they pose on people living nearby. Looking at the answer options, A is incorrect, as although mobile risk is mentioned as suspected, it is not said to be fact, therefore can not be an accurate conclusion. Options B and D are also incorrect for similar reasons, as they conclude that those living close to the mast must be over estimating the risk, yet as we are unable to evaluate the risk accurately in the text we can not conclude this either. Looking at answer C, it is incorrect, as the passage states there is less radiation from mobile phones than masts. Therefore, answer option E must be correct, it takes into account the evaluation of the risk and public perception, therefore being an accurate conclusion of the passage.

    Post Comment

    University education is not a matter of forcing through the largest number of graduates at the lowest possible short-term cost. This would not serve the needs of the current generation of students and, even from a narrow economic viewpoint, it would not promote economic prosperity in the long term. It is often a poor economic practice to skimp on the production costs of a product, even when you are only interested in the eventual monetary profit to be made from it. But in any event university education has wider purposes than merely the promotion of economic prosperity.

    40. Which of the following is an underlying assumption of the above argument?
  • 0
    0

    Explanation

    The correct answer is D.

    Post Comment
    anon Medicmind Tutor

    Wed, 27 Sep 2023 08:34:04

    why is the answer D?

    The preservation of First Class sections on trains is indefensible. This is not because it is socially divisive, which it is; nor because it is very expensive, which it also is; but because it inevitably leads to a relatively poorer service in Standard Class sections of trains. Clearly, in order to attract First Class passengers it is necessary to maintain certain levels of discomfort in those compartments which cater to the great majority of travellers. There is, for example, a powerful disincentive to make more space generally available on trains, as one of the main inducements to travel First Class is to escape overcrowding and be assured of a seat. Yet, additional space would add significantly to the safety and comfort of most passengers.

    41. Which one of the following, if true, would most weaken the above argument?
  • 0
    0

    Explanation

    The correct answer is B.

    The argument is that there is no reason to argue to keep first class sections on trains. Answer B most weakens the argument by giving a valid reason for keeping first class seats, as the revenue they generate helps the whole rail service. A does not weaken the argument because just because people are using first class sections, doesn’t mean it would be better to keep them than not. The amount of space they take up is irrelevant, as the argument is that their existence is indefensible, hence eliminate C. Similarly, D is irrelevant. E may weaken the argument by giving a reason for keeping the first class sections, but B weakens the argument to a greater extent because it gives a benefit of the whole rail service.

    Post Comment

    The piece of card shown below is folded to make a box with a triangular cross-section.

    42. Which one of the views below can not be of this box?
  • 0
    0

    Explanation

    The correct answer is E.

    A is possible as the rectangular face shown in A is the rectangle at the top of the card in the diagram given in the question and is adjacent to the triangle on the right hand side of the diagram. B similarly shows the rectangular face at the bottom of the diagram. C shows the middle rectangular face, and the triangular face on the right hand side of the diagram in the question. D is like A but includes the triangular face on the left of the diagram (rather than the one on the right shown in A). Therefore E must not be possible.

    Post Comment

    At Marland College, all 120 pupils in year 11 take two languages at GCSE. The table below shows the numbers of pupils taking each language combination.

    43. What percentage of those taking Spanish are also taking French? (Give your answer to the nearest 1% if necessary).
  • 0
    0

    Explanation

    The correct answer is C.

    First calculate the total number of pupils taking Spanish by adding up all the numbers in sections of the table which are either in the Spanish row or column: 14+9+13+4=40. Then find the section of the table which corresponds to both Spanish and French, in which the number is 14. So the proportion of students taking Spanish that are also taking French would be: 14/40=7/20=3.5/10=0.35=35%.

    Post Comment

    The diagram below shows Morden Hall and the network of paths within the grounds.

    When the owner takes his dog for a walk he always starts and finishes at the intersection outside the front door and (keeping to the paths) walks one complete triangular route.

    44. How many different routes does he have to choose from? (Clockwise and anticlockwise around the same triangle counts as one route only.)
  • 0
    0

    Explanation

    The correct answer is D.

    If we give each triangle a name: The top triangle is triangle 1, the two triangles below are L and R triangle 2 and the two triangles at the bottom are L and R triangle 3. There are 5 separate triangles in total, each of which touch the intersection, therefore each is a different possible path that could be taken, giving 5 possible paths so far. Now we need to find triangular paths that can be made by joining two or more triangles together. The following combinations of triangles are possible: 1+L2, 1+R2, L2+L3, R2+R3, L2+L3+R3, R2+R3+L3, giving us 6 more possible paths. So the total number of possible paths are: 5+6=11.

    Post Comment

    Although study leave in the run up to exams is traditional, and benefits well-motivated, organised students, recent thinking suggests that it is not the best way to improve results. Some students are required to do housework or help in the family business rather than studying. Many others spend time playing computer games, watching daytime TV or hanging around with friends instead of revising. Schools which provide in-school study areas with teacher supervision and a programme of booster revision sessions are getting more A*s and fewer Ds. It is time to ditch study leave.

    45. Which one of the following best expresses the main conclusion of the argument?
  • 0
    0

    Explanation

    The correct answer is D.

    The passage talks about the benefits and disadvantages of study leave. It speaks about how hard working students are benefited by this, but this is not the cases for all students for a variety of reasons outlined. The argument rounds by saying that schools providing in-school studying areas with a revision programme produces better results, concluding that we should abandon study leave. Looking at the answer options, A, B, C and E are all points raised in the text and all help to support the valid conclusion, but are not the overall conclusion on their own. While answer option D reflects the overall tone of the argument and is the supported conclusion by all the other answer options, therefore it must be the conclusion.

    Post Comment

    Land reform is urgently needed in the UK. It is well documented that around two-thirds of Britain is owned by just 0.3% of the population. Many of the largest estates have been in the hands of the same families for centuries, and were acquired under outdated feudal laws, often by means of aggression. If these families had any sense of social justice in the 21st century they would by now have voluntarily surrendered all but the acreage they needed for personal privacy. __________________________________ the state should confiscate all excessive land holdings without compensation and redistribute them into the families of workers and taxpayers at affordable prices.

    46. Which of the following phrases, inserted in the blank space, most logically completes the above argument?
  • 0
    1

    Explanation

    The correct answer is E.

    Post Comment
    anon Medicmind Tutor

    Thu, 05 Oct 2023 09:19:39

    why is it E?

    All trees have leaves. An oak is a tree. Therefore oaks have leaves.

    47. Which one of the following most closely parallels the reasoning used in the argument?
  • 0
    0

    Explanation

    The correct answer is A.

    For the argument given in the question: All X have Y. Z is an X. Therefore Z has Y. (where X=trees, Y=leaves and Z=oaks). In answer A, X=leaves, Y=trees and Z=that plant.

    Post Comment

    Megan pays a subscription to Probe Information Services. This is an internet service which allows her to access up-to-date information relevant to her job. The leaflet which came with her application form is reproduced below.

    Megan’s call charges are at local rates of 10c for up to 2 minutes and 5c per additional minute or part of a minute. In the first month, Megan seeks detailed information on 5 companies and uses the other services a total of 42 times.

    48. How much will Probe bill her?
  • 0
    0

    Explanation

    The correct answer is B.

    Cost for seeking detailed information: 50c x 5 enquiries =250c. Cost for other services: 10c x 42 enquiries =420c. In total: 250+420=670c=$6.70.

    Post Comment

    A group of Construction students must choose their specialist options from the following list:

    Bricklaying, Damp-proofing, Drainage, Flooring, Joinery, Plastering, Roofing

    Each student must choose three options. The available options may be combined with each other in any way, with the exception of the restrictions that Damp-proofing and Drainage together may not be combined with Bricklaying or Plastering because of timetable constraints, and that students choosing Joinery must also choose Flooring.

    49. How many possible combinations including Drainage could the students choose from?
  • 0
    2

    Explanation

    The correct answer is C.

    Let bricklaying=B, Damp-proofing=Dp, drainage=D, flooring=F, joinery=J, plastering=P, roofing=R.

    Post Comment
    U Medicmind Tutor

    Mon, 24 Jul 2023 16:12:43

    Is there a more clear explanation on the process

    anon Medicmind Tutor

    Thu, 05 Oct 2023 09:20:43

    why is it C?

    I am a member of a book club. Every month I must choose at least one book. All books are sold at a discount of £2.00 on shop prices. I pay a fixed charge for postage regardless of which books I order.

    50. Which one of the following graphs could show the relationship between the number of books ordered in a month and the total saving on shop prices (allowing for postage charge)?
  • 0
    0

    Explanation

    The correct answer is C.

    Post Comment
    anon Medicmind Tutor

    Tue, 26 Sep 2023 11:50:58

    why does buying one book not give you savings of £2?

    James Medicmind Tutor

    Tue, 10 Oct 2023 17:08:11

    I think because the postal charge must be $2 - this is the trick. That is why it says 'could show' - all the others seem more likely but have little errors meaning they are unlikely, whereas C seems unlikely but in this unlikely scenario it actually works

    2011 S1 Review Screen

    Instructions

    Below is a summary of your answers. You can review your questions in three (3) different ways.

    The buttons in the lower right-hand corner correspond to these choices:

    1. Review all of your questions and answers.
    2. Review questions that are incomplete.
    3. Review questions that are flagged for review. (Click the 'flag' icon to change the flag for review status.)

    You may also click on a question number to link directly to its location in the exam.

    2011 S1 Section

    Final Answer Review Screen

    Instructions

    This review section allows you to view the answers you made and see whether they were correct or not. Each question accessed from this screen has an 'Explain Answer' button in the top left hand side. By clicking on this you will obtain an explanation as to the correct answer.

    At the bottom of this screen you can choose to 'Review All' answers, 'Review Incorrect' answers or 'Review Flagged' answers. Alternatively you can go to specific questions by opening up any of the sub-tests below.

    2011 S1 Section

    x

    +/-
    %
    MRC
    M-
    M+
    7
    8
    9
    4
    5
    6
    1
    2
    3
    ON/C
    0
    .
    ÷
    ×
    -
    +
    =

    Let's get acquainted ?
    What is your name?

    Next

    Nice to meet you, {{name}}!
    What is your preferred phone number?

    What is your preferred phone number?

    Next

    Just to check, what are you interested in?

    1-1 Tutoring Online Course Bursaries/Resources Other

    When should we call you?

    Another Day
    Skip the Call

    What time works best for you? (UK Time)

    8am-2pm 2pm-10pm
    10:00-10:30 10:30-11:00 11:00-11:30 11:30-12:00 12:00-12:30 12:30-13:00
    15:00-15:30 15:30-16:00 16:00-16:30 16:30-17:00 17:00-17:30 17:30-18:00 18:00-18:30 18:30-19:00 19:00-19:30 19:30-20:00

    How many hours of 1-1 tutoring are you looking for?

    0-5 10 20-30 40+

    My WhatsApp number is...

    Same as the one I entered Different to the one I entered

    For our safeguarding policy, please confirm...

    For our safeguarding policy, please confirm...

    I am under 18 I am over 18

    Which online course are you interested in?

    Next

    What is your query?

    Submit

    Sure, what is your query?

    Submit

    Loading...

    Thank you for your response.
    We will aim to get back to you within 12-24 hours.

    Lock in a 2 Hour 1-1 Tutoring Lesson Now

    If you're ready and keen to get started click the button below to book your first 2 hour 1-1 tutoring lesson with us. Connect with a tutor from a university of your choice in minutes. (Use FAST5 to get 5% Off!)

    Buy Now for £70